4
$\begingroup$

I want to prove $\sup⁡(A\cup B)=\max\{\sup⁡(A),\sup⁡(B)\}$, where $A,B\subset \mathbb R$ are non-empty and bounded sets from above. I have reviewed similar questions and answers, but I intended to prove directly using the following definition of least upper bound:

$\sup ( A) \in \mathbb R$ and $A\subseteq \mathbb R$ if

  1. $\forall a\in A$, $a \leq \sup( A)$,
  2. $\forall \varepsilon >0$, $\exists b \in A$ such that $\sup( A)-\varepsilon<b$.

Here's my proof:

  1. We know $A \cup B=\{x \mid x\in A \text{ or }x\in B \} $,
  2. $\forall a \in A$, $a \leq \sup⁡(A) $ and $\forall b \in B$, $b \leq \sup⁡(B) $, thus $\forall a \in A$ and $\forall b \in B$, $a,b\leq \max\{\sup⁡(A),\sup⁡(B)\}$.
  3. $\forall \varepsilon>0 $, $\exists k_A \in A$ s.t. $\sup( A)-\varepsilon <k_A$ and $\exists k_B \in B$ s.t. $\sup( B)-\varepsilon <k_B$. $k_A\in A\cup B$, and $\sup(A)\geq \sup(B)$ implies $\sup(A)-\varepsilon=\max\{\sup⁡(A),\sup⁡(B)\}-\varepsilon <k_A$. After considering vice versa, we can write, $\exists k\in A\cup B$, $\max\{\sup⁡(A),\sup⁡(B)\}-\varepsilon<k$.

Since both properties are satisfied, we can conclude that $\sup⁡(A\cup B)=\max\{\sup⁡(A),\sup⁡(B)\}$.

Question: Is this proof correct? I have some bad feelings regarding my 3rd argument. Thank you!

$\endgroup$
12
  • 1
    $\begingroup$ Looks good to me! As a small recommendation, in an argument like this it's sometimes clearer to assume wlog that $\sup A \geq \sup B$ $\endgroup$
    – jpmacmanus
    Commented Jul 12 at 13:30
  • 2
    $\begingroup$ You are right to have "bad feelings regarding 3rd argument". Your "let $k\in A\cup B$" is really unclear. Who is this $k$? Is it $k_A$? or $k_B$? or someone else? Instead, assuming $\sup A\ge\sup B$, choose $k=k_A$. $\endgroup$ Commented Jul 12 at 13:31
  • 1
    $\begingroup$ @JMP The question is different. OP is asking for a proof verification. $\endgroup$ Commented Jul 12 at 13:34
  • 1
    $\begingroup$ Your proof will then become ok, but only if $A,B$ are non-empty and bounded from above. For other proofs, see math.stackexchange.com/questions/921975 provided by JMP, and the many posts linked to it. $\endgroup$ Commented Jul 12 at 13:52
  • 1
    $\begingroup$ You have to start with the fact that both $A$ and $B$ are bounded. Then you should show that $A\cup B$ is bounded and then proceed to the result. $\endgroup$
    – John Douma
    Commented Jul 12 at 14:35

2 Answers 2

1
$\begingroup$

Your proof looks fine after following the suggestions in the comments. But you may consider writing it in plain English to make it easier to understand.

Let $A, B \subseteq \mathbb{R}$ be bounded from above; i.e., there is some $x \in \mathbb{R}$ such that $a < x$ for all $a \in A$ and some $y \in \mathbb{R}$ such that $b < y$ for all $b \in B$. Let $C = A \cup B$.

The set $C$ must be bounded from above: take $z = \max\{a, b\} \in \mathbb{R}$; for any $c \in C$, if $c \in A$, then $c < x \le z$ and if $c \in B$, then $c < y \le z$.

Denote $m = \max\{\sup A, \sup B\}$; we want to show that $m = \sup C$. Then (1) $m$ needs to be an upper bound of $C$, and (2) $m$ cannot be larger than any upper bound of $C$ (equivalent to the second part of your definition).

To show (1) is easy: let $c \in C$ be arbitrary. If $c \in A$, then $c \le \sup A \le m$ and if $c \in B$, then $c \le \sup B \le m$. Either way, $c \le m$, as desired.

To show (2), suppose for the sake of contradiction that there is some $n < m$ that is an upper bound of $C$. Then $n < \sup A$ or $n < \sup B$. In the former case, there is some $a \in A$ such that $a > n$; since $a \in C$, this contradicts the fact that $n$ is an upper bound of $C$. A similar argument applies if $n < \sup B$. Therefore, $m$ must be the smallest upper bound of $C$, completing the proof.

$\endgroup$
0
$\begingroup$

Option:

$1)A \subset (A\cup B)$, and

$B\subset (A\cup B),$ hence

$\sup (A) \le \sup (A\cup B)$, and $\sup (B) \le \sup (A\cup B) $.

Thus

$\max ${$\sup (A) , \sup (B) $} $\le \sup (A\cup B).$

$2)a\le \max ${$\sup (A) , \sup (B) $}, and
$b\le \max ${$\sup (A) ,\sup (B) $},

Let $c \in (A \cup B),$ then
$c \le \max ${$\sup (A), \sup (B)$}, i e.

$\max ${$\sup(A),\sup (B) $}
is an upper bound for $c \in (A\cup B) $.

Since $\sup (A\cup B)$ is the least upper bound for {$c$}, we have

$\sup (A \cup B)$
$\le \max ${$\sup (A), \sup (B)$},

and we are done.

$\endgroup$
4
  • $\begingroup$ Thanks for your proof! I am hesitant to use $\sup(A) \leq \sup(A \cup B)$ in proving this property because it seems to rely on a property of equal level. (I don't know if that makes sense). That is why I preferred to prove it directly from the definition of $\sup$ only. $\endgroup$
    – Ba_nanza
    Commented Jul 14 at 8:28
  • $\begingroup$ @Ba_nanza. Property of equal level I am not familiar with this. Does it make sense if you have $ D \subset E$ then sup (D) $\le$ sup (E) ? $\endgroup$ Commented Jul 14 at 8:56
  • $\begingroup$ Sure, that makes sense. Although this is a very elementary proof and property, the proof you use requires another relation that seems to need the property "$A \subset B \Rightarrow \sup(A) \leq \sup(B)$," which also needs a proof of similar complexity to the one we are trying to make. That is why I aimed to use a proof that relies only on even more primitive relations (in this case, only the definitions of $\sup$). $\endgroup$
    – Ba_nanza
    Commented Jul 14 at 11:43
  • $\begingroup$ That is fine, you proved it. $\endgroup$ Commented Jul 14 at 12:06

You must log in to answer this question.

Not the answer you're looking for? Browse other questions tagged .